Difference between revisions of "1983 AHSME Problems/Problem 30"
MRENTHUSIASM (talk | contribs) m (→Problem) |
J314andrews (talk | contribs) (→Solution) |
||
(One intermediate revision by one other user not shown) | |||
Line 38: | Line 38: | ||
\textbf{(E)}\ 30^{\circ} </math> | \textbf{(E)}\ 30^{\circ} </math> | ||
− | ==Solution== | + | == Solution 1 == |
Since <math>\angle CAP = \angle CBP = 10^\circ</math>, quadrilateral <math>ABPC</math> is cyclic (as shown below) by the converse of the theorem "angles inscribed in the same arc are equal". | Since <math>\angle CAP = \angle CBP = 10^\circ</math>, quadrilateral <math>ABPC</math> is cyclic (as shown below) by the converse of the theorem "angles inscribed in the same arc are equal". | ||
Line 76: | Line 76: | ||
Since <math>CA = CB</math>, triangle <math>ABC</math> is isosceles, with <math>\angle BAC = \angle ABC = 30^\circ</math>. Now, <math>\angle BAP = \angle BAC - \angle CAP = 30^\circ - 10^\circ = 20^\circ</math>. Finally, again using the fact that angles inscribed in the same arc are equal, we have <math>\angle BCP = \angle BAP = \boxed{\textbf{(C)}\ 20^{\circ}}</math>. | Since <math>CA = CB</math>, triangle <math>ABC</math> is isosceles, with <math>\angle BAC = \angle ABC = 30^\circ</math>. Now, <math>\angle BAP = \angle BAC - \angle CAP = 30^\circ - 10^\circ = 20^\circ</math>. Finally, again using the fact that angles inscribed in the same arc are equal, we have <math>\angle BCP = \angle BAP = \boxed{\textbf{(C)}\ 20^{\circ}}</math>. | ||
+ | |||
+ | == Solution 2 == | ||
+ | <asy> | ||
+ | import geometry; | ||
+ | import graph; | ||
+ | |||
+ | unitsize(4 cm); | ||
+ | |||
+ | pair A, B, C, M, N, P, Q, R; | ||
+ | |||
+ | M = (-1,0); | ||
+ | N = (1,0); | ||
+ | C = (0,0); | ||
+ | A = dir(140); | ||
+ | B = dir(20); | ||
+ | Q = dir(-20); | ||
+ | R = dir(-40); | ||
+ | P = extension(A, A + rotate(10)*(C - A), B, B + rotate(10)*(C - B)); | ||
+ | |||
+ | draw(M--N); | ||
+ | draw(circle(C,1)); | ||
+ | draw(A--C--B); | ||
+ | draw(A--P--B); | ||
+ | draw(A--Q--C); | ||
+ | draw(A--R--C); | ||
+ | draw(A--B); | ||
+ | draw(B--Q); | ||
+ | |||
+ | label("$A$", A, NW); | ||
+ | label("$B$", B, E); | ||
+ | label("$C$", C, SW); | ||
+ | label("$M$", M, SW); | ||
+ | label("$N$", N, SE); | ||
+ | label("$P$", P, S); | ||
+ | label("$Q$", Q, SE); | ||
+ | label("$R$", R, SE); | ||
+ | </asy> | ||
+ | |||
+ | Complete the circle centered at <math>C</math> and extend <math>AP</math> and <math>AC</math> to meet the circle at <math>Q</math> and <math>R</math>, respectively. So <math>\stackrel{\frown}{NR} = \stackrel{\frown}{MA} = 40^\circ</math>, and <math>\stackrel{\frown}{QR} = 2 \cdot 10^\circ = 20^\circ</math>. Therefore, <math>\stackrel{\frown}{NQ} = 40^\circ - 20^\circ = 20^\circ</math>. | ||
+ | |||
+ | Since <math>CA = CQ</math>, <math>\triangle CAQ</math> is isosceles and <math>\angle CQA = \angle CAQ = 10^\circ</math>. Let <math>\theta = \angle CBQ = \angle CQB</math>. Then <math>\angle PBQ = \angle PQB = \theta - 10^\circ</math>, so <math>\triangle PBQ</math> is isosceles with <math>PB = PQ</math>. So <math>CN</math> must bisect <math>\angle BCQ</math>, and <math>\stackrel{\frown}{BN} = \stackrel{\frown}{NQ} = \boxed{(\mathbf{C})\ 20^{\circ}}</math>. | ||
+ | |||
+ | -j314andrews | ||
+ | |||
+ | == Video Solution == | ||
+ | |||
+ | https://youtu.be/fZAChuJDlSw?si=wJUPmgVRlYwazauh | ||
+ | |||
+ | ~ smartschoolboy9 | ||
==See Also== | ==See Also== |
Latest revision as of 20:32, 2 July 2025
Problem
Distinct points and
are on a semicircle with diameter
and center
.
The point
is on
and
. If
, then
equals
Solution 1
Since , quadrilateral
is cyclic (as shown below) by the converse of the theorem "angles inscribed in the same arc are equal".
Since ,
, so, using the fact that opposite angles in a cyclic quadrilateral sum to
, we have
. Hence
.
Since , triangle
is isosceles, with
. Now,
. Finally, again using the fact that angles inscribed in the same arc are equal, we have
.
Solution 2
Complete the circle centered at and extend
and
to meet the circle at
and
, respectively. So
, and
. Therefore,
.
Since ,
is isosceles and
. Let
. Then
, so
is isosceles with
. So
must bisect
, and
.
-j314andrews
Video Solution
https://youtu.be/fZAChuJDlSw?si=wJUPmgVRlYwazauh
~ smartschoolboy9
See Also
1983 AHSME (Problems • Answer Key • Resources) | ||
Preceded by Problem 29 |
Followed by Last Problem | |
1 • 2 • 3 • 4 • 5 • 6 • 7 • 8 • 9 • 10 • 11 • 12 • 13 • 14 • 15 • 16 • 17 • 18 • 19 • 20 • 21 • 22 • 23 • 24 • 25 • 26 • 27 • 28 • 29 • 30 | ||
All AHSME Problems and Solutions |
These problems are copyrighted © by the Mathematical Association of America, as part of the American Mathematics Competitions.